Strengthen Questions - - Question 5
Despite improvements in treatment for asthma, the death rate from this disease has doubled during the past decade fro...
Replies
Naz June 30, 2014
Here we are faced with a Bizarro Strengthen question. Our correct answer will not strengthen the argument, meaning it will either weaken it or do nothing (i.e. irrelevant).First, let's break down the argument. The conclusion is: "the cause of increased deaths is the use of bronchial inhalers by asthma sufferers to relieve their symptoms."
Premise: "Despite improvements in treatment for asthma, the death rate from this disease has doubled during the past decade from its previous rate." We are also told that "the rate of deaths due to asthma has increased dramatically even in cities with long-standing, comprehensive medical records and with little or no urban pollution."
So the argument presents us with two possible explanations (i.e. more widespread and accurate records and increase in urban pollution) of why the death rate from asthma has doubled during the past decade, despite improvements in its treatment. Then it gives reasons why those two explanations are actually without merit and concludes that the increase must be due to the use of bronchial inhalers by asthma sufferers.
Answer choice (A) is the only answer choice that does not strengthen the argument. Urban populations doubling in the past decade is irrelevant to the argument because the RATE of the disease has doubled. Regardless of the number of people living in urban populations, the percentage of people with asthma has still doubled.
Remember, the correct answer in a Bizarro Strengthen question is going to either weaken the answer choice or do nothing. In this case, the rate of deaths has increased, so the population count does not matter. So, answer choice (A) has no bearing on the argument, i.e. it is our correct answer.
Hope that was helpful! Let us know if you have any other questions.
cupton22 December 3, 2017
Can you explain why B is not correct?Thanks.
Kuma June 30, 2018
I would also like to know why B is incorrect.
Mehran July 1, 2018
Hi @Kuma! Thanks for your post. Please be sure to refer to our previous posts on this thread and be clear on the stimulus (that it's an argument; what the conclusion is; and what the premises are that are given in support of that conclusion).This is a Bizarro Strengthen question, so the four incorrect answers will strengthen the argument in the stimulus.
Answer choice (B) does strengthen the conclusion that the increased death rates are due to a cause other than improved record keeping; this answer strengthens the argument by bolstering one of the premises in the stimulus ("since the rate of deaths due to asthma has increased dramatically even in cities with long-standing, comprehensive medical records").
Strengthening this point helps to rule out improved record keeping as the cause of the perceived effect (the doubling of the death rate). In turn, this supports the conclusion that an entirely different cause is responsible - the use of bronchial inhalers.
Hope this helps! Please let us know if you have any additional questions.
Adam-Estacio March 25, 2019
I don't see how answer choice B actually strengthens the argument. It says record keeping is just "as accurate", not that it has improved at all. Answer choice A seems to be the better answer because if the urban population has doubled, the amount of urban pollution would have become worse, thus supporting one of the stated causes (increased urban population).